Đến nội dung

Hình ảnh

Đề thi Chọn Đội tuyển Dự thi HSG Quốc Gia Đà Nẵng 2014-2015


  • Please log in to reply
Chủ đề này có 14 trả lời

#1
thukilop

thukilop

    Thượng sĩ

  • Thành viên
  • 291 Bài viết

SỞ GIÁO DỤC VÀ ĐÀO TẠO THÀNH PHỐ ĐÀ NẴNG

ĐỀ THI CHỌN ĐỘI TUYỂN QUỐC GIA 2014-2015

 

VÒNG 1 (11/9/2014)

$\boxed{\text{Bài 1 (5đ)}}$

 Tìm công thức tính số hạng tổng quát của dãy số $(x_n)$ biết:

$x_1=\frac{2013}{2014}$,$x_{n+1}=\frac{1}{4+2011x_n}$ (với mọi $n>0$)

 

Chứng minh dãy số $(x_n)$ có giới hạn hữu hạn. Tìm giới hạn đó

$\boxed{\text{Bài 2 (5đ)}}$

 

Tìm tất cả các hàm số $f: \mathbb{Z} \to \mathbb{R}$ sao cho $f(0)\neq 0$,$f(1)=6$ và

$$f(x)f(y)=f(x+y)+f(x-y)$ với mọi $x,y\in \mathbb{Z}$$

 

$\boxed{\text{Bài 3 (5đ)}}$

 

Cho hai đường tròn $(C_1)$ và $(C_2)$ cắt nhau tại hai điểm phân biệt C,D sao cho tâm O của $(C_2)$ nằm trên $(C_1)$. Gọi A là điểm trên $(C_1)$ và B là điểm nằm trên $(C_2)$ sao cho đường thẳng AC tiếp xúc với $(C_2)$ tại C và đường thẳng BC tiếp xúc với $(C_1)$ tại C. Đường thẳng AB cắt lại $(C_2)$ tại E và cắt $(C_1)$ tại F. Gọi G là giao điểm thứ hai của đường thẳng CE và $(C_1)$. Hai đường thẳng CF và GD cắt nhau tại H. Chứng minh rằng giao điểm của GO và EH là tâm đường tròn ngoại tiếp tam giác DEF

 

$\boxed{\text{Bài 4 (5đ)}}$

 

Tại một hội nghị quốc tế, các thành viên tham dự đều biết ít nhất một trong ba thứ tiếng: Anh, Pháp, Đức. Biết rằng số thành viên biết Tiếng Anh, số thành viên biết Tiếng Pháp và số thành viên biết Tiếng Đức cùng bằng 50. Chứng minh rằng có thể chia tất cả các thành viên tham dự hội nghị thành 5 nhóm sao cho trong mỗi nhóm có đúng 10 thành viên biết tiếng Anh, đúng 10 thành viên biết tiếng Pháp và đúng 10 thành viên biết tiếng Đức.

 

VÒNG 2 (12/9/2014)

$\boxed{\text{Bài 5 (7đ)}}$

 

Cho tam giác ABC nhọn không cân có O là tâm ngoại tiếp. Gọi P là một điểm nằm trong tam giác sao cho AP vuông góc với BC. Đường trung trực của đoạn AP cắt AC tại M. Đường trung trực của đoạn thẳng MC cắt BC tại N, các đường thẳng AO và MN cắt nhau tại K. Gọi D là điểm đối xứng của O qua BC

a) Chứng minh rằng đường thẳng AD đi qua trung điểm Q của đoạn thẳng PK.

b) Gọi E và F lần lượt là hình chiếu vuông góc của P lên CA và AB. Chứng minh rằng đường trung trực của đoạn thẳng EF đi qua Q.

c) Gọi I là trung điểm của đoạn thẳng BC. Đường trung trực của đoạn thẳng EF cắt đường thẳng AI tại T. Chứng minh KT vuông góc BC

 

$\boxed{\text{Bài 6 (7đ)}}$

 

Với mỗi số nguyên dương n, gọi $f(n)$ là số cách thay các dấu $"\pm "$ trong biểu thức $\pm 1\pm 2\pm 3...\pm n$ bởi các dấu $"+"$ hoặc $"-"$ sao cho tổng đại số nhận được bằng 0. Chứng minh rằng:

a) $f(n)=0$ khi $n\equiv 1 (mod 4)$ hoặc $n\equiv 2 (mod 4)$

b)$2^{\frac{n}{2}-1}\leq f(n)<2^n-2^{\left [ \frac{n}{2} \right ]+1}$ khi $n\equiv 0 (mod 4)$ hoặc $n\equiv 3 (mod 4)$

 

$\boxed{\text{Bài 7 (6đ)}}$

 

Các ô vuông của một bảng vuông kích thước $10\times 10$ được tô bởi các màu trắng hoặc đen sao cho trên mỗi hàng cũng như trên mỗi cột đều có đúng 3 ô được tô màu đen. Chẳng hạn như hình vẽ :

                                                                                10644932_634592509991520_663527048267035

Chứng minh rằng trong mọi cách tô như vậy ta luôn có thể tìm ra 10 ô được tô màu đen sao cho không có hai ô nào nằm trên cùng một hàng hay trên cùng một hàng cột.

=======Hết=======


-VƯƠN ĐẾN ƯỚC MƠ-


#2
perfectstrong

perfectstrong

    $LOVE(x)|_{x =\alpha}^\Omega=+\infty$

  • Quản lý Toán Ứng dụng
  • 4991 Bài viết

Bài 3:

Gọi $O'$ là tâm của đường tròn $(C_1)$. $X$ là giao của $GO$ và $HE$.

Vì $\angle ACO=90^o \Rightarrow AO$ là đường kính của $(C_1)$ nên $O'$ là trung điểm $AO$.

Mà ta lại có: $(CA,CO')\equiv(CO,CB) \pmod{\pi}$ (do $CA \perp CO, CO' \perp CB$) và $\triangle CO'A,COB$ là các tam giác cân tại $O,O'$ tương ứng nên $\triangle CO'A,COB$ đồng dạng nghịch

Nên suy ra\[\left( {EC,EB} \right) \equiv \frac{1}{2}\left( {\overrightarrow {OC} ,\overrightarrow {OB} } \right) \equiv \frac{1}{2}\left( {\overrightarrow {O'C} ,\overrightarrow {O'A} } \right) \equiv \left( {FC,FA} \right)\pmod{\pi}\]

Do đó $\triangle CEF$ cân tại $C$. (1)

Chú ý rằng $C,D$ đối xứng qua $OO'$ nên ta có $(EA,ED) \equiv (EB,ED) \equiv (CB,CD) \equiv (O'C,O'A) \equiv (O'A,O'D) \pmod{\pi}$. Do đó $A,O',E,D$ đồng viên (2)

Suy ra $(O'O,O'E) \equiv (DA,DE) \equiv (CD,CE) \pmod{\pi}$. Mà $O'O \perp CD \Rightarrow O'E \perp CE$.

Nên $E$ là trung điểm $CG$ (3)

$CEDB$ là tứ giác điều hòa nên $CD$ là đường đối trung của $\triangle ECB \Rightarrow (CE,CF) \equiv (CD,CB) \equiv (GC,GD) \pmod{\pi} \Rightarrow HCF$ cân tại $H$.

Từ đó $HX$ là phân giác $\angle GHC$. Mà $OC=OD \Rightarrow GO$ là phân giác $\angle HCD \Rightarrow X$ là tâm đường tròn nội tiếp $\triangle HCD$.

Lại có $CE=CF=\dfrac{CG+CH-GH}{2}=GE \Rightarrow E,F,D$ là tiếp điểm của $(X)$ trên các cạnh $\triangle HCD$. Ta có đpcm.

170914.png

 

Bài 5:

a) Dựng đường kính $AA'$. Đường thẳng qua $H$ song song $BC$ cắt $AC$ tại $X$.

Trước hết, ta chứng minh nhận xét sau: $(XA',XC) \equiv (CA,CB) \pmod{\pi}$

Thật vậy, vẽ $BH$ cắt $(O)$ lần 2 tại $Y$. Dễ thấy $H, Y$ đối xứng qua $AC \Rightarrow (YX,YA) \equiv \dfrac{\pi}{2} \pmod{\pi}$

Mà $AA'$ là đường kính $\Rightarrow (YA',YA) \equiv \dfrac{\pi}{2} \pmod{\pi} \Rightarrow Y,X,A'$ thẳng hàng.

$\Rightarrow (YA',YC) \equiv (YX,YA) \equiv (YA,YH) \equiv (CA,CB) \pmod{\pi}$

170914 2.png

Lấy $Z$ là trung điểm $AX \Rightarrow ZO \parallel XA' \Rightarrow (ZO,ZC) \equiv (XA',XC) \equiv (CA,CB) \equiv (MN,MC) \pmod{\pi} \Rightarrow MN \parallel OZ$

Vì thế \[\frac{{\overline {AK} }}{{\overline {AO} }} = \frac{{\overline {AM} }}{{\overline {AZ} }} = \frac{{\overline {AP} }}{{2\cos \left( {\overrightarrow {AP} ,\overrightarrow {AM} } \right)}}:\frac{{\overline {AH} }}{{2\cos \left( {\overrightarrow {AH} ,\overrightarrow {AZ} } \right)}} = \frac{{\overline {AP} }}{{\overline {AH} }}\]

Nên $PK \parallel HO$. Ta đã biết $AD$ đi qua trung điểm $HO$ nên $AD$ cũng qua trung điểm $Q$ của $PH$.

 

b) Gọi đường cao $BB',CC'$ của $\triangle ABC$. Đặt $k=\dfrac{\overline{AP}}{\overline{AH}}$. Gọi $R$ là trung điểm $OH$.

\[\frac{{\overline {AC'} }}{{\overline {AF} }} = \frac{{\overline {AH} }}{{\overline {AP} }} = \frac{{\overline {AB'} }}{{\overline {AE} }}\]

Xét phép vị tự tâm $A$, tỉ số $k$:\[V_A^k:\left\{ \begin{array}{l}
H \mapsto P\\
O \mapsto K\\
C' \mapsto F\\
B' \mapsto E
\end{array} \right. \Rightarrow \left\{ \begin{array}{l}
B'C' \to EF\\
R \mapsto Q
\end{array} \right.\]

Mà $R$ thuộc trung trực $EF$ nên $Q$ thuộc trung trực $B'C'$.

170914 3.png

 

c) $RB'=RC', IB'=IC' \Rightarrow IR$ là trung trực $B'C'$. Gọi $d$ là trung trực $EF$.

Xét $V_{A}^k$: theo câu b thì $IR \to d$

Mà $AI \to AI \Rightarrow I=AI \cap IR \mapsto AI \cap d=T \Rightarrow OI \to KT \Rightarrow OI \parallel KT$

Vì $OI \perp BC \Rightarrow KT \perp BC$


Bài viết đã được chỉnh sửa nội dung bởi perfectstrong: 17-09-2014 - 12:59

Luôn yêu để sống, luôn sống để học toán, luôn học toán để yêu!!! :D
$$\text{LOVE}\left( x \right)|_{x = \alpha}^\Omega = + \infty $$
I'm still there everywhere.

#3
Juliel

Juliel

    Thượng úy

  • Thành viên
  • 1240 Bài viết

$\boxed{\text{Bài 2 (5đ)}}$

 

Tìm tất cả các hàm số $f: Z ---> R$ sao cho $f(0)\neq 0$,$f(1)=6$ và

$f(x)f(y)=f(x+y)+f(x-y)$ với mọi $x,y\in \mathbb{Z}$

 

Lời giải :

Trong $(1)$ cho $x=y=0$ được :

$$f^2(0)=2f(0)\Rightarrow f(0)=2$$

Trong $(1)$ cho $x=0$ :

$$2f(y)=f(y)+f(-y),\;\forall y\in \mathbb{Z}\Rightarrow f(x)=f(-x),\;\forall x\in \mathbb{Z}$$

Vậy ta chỉ cần tìm $f$ trên $\mathbb{N}$ là đủ.

Trong $(1)$ cho $x=n\in \mathbb{N},y=1$ :

$$f(n+1)+f(n-1)=6f(n),\;\forall n \in \mathbb{N}$$

Đặt $x_n=f(n)$ ta được dãy $(x_n)$ :

$$\left\{\begin{matrix} x_0=2,x_1=6\\ x_{n+1}=6x_n-x_{n-1},\;\forall n=0,1,2,... \end{matrix}\right.$$

Công việc còn lại là giải phương trình sai phân này.

 

Bài hình ngày 1 hình như xuất hiện trên THTT rồi, hình như do thầy Thông đề nghị luôn thì phải =))


Bài viết đã được chỉnh sửa nội dung bởi Juliel: 17-09-2014 - 11:26

Đừng rời xa tôi vì tôi lỡ yêu người mất rồi !
 

Welcome to My Facebook !


#4
TonnyMon97

TonnyMon97

    Trung sĩ

  • Thành viên
  • 124 Bài viết

Bài 1: Bằng qui nạp dễ dàng chứng minh $0< x_n <1 \forall n\ge 0       (*)$

Ta cần chứng minh dãy $x_n$ đơn điệu tăng hoặc giảm.

Xét hàm số $f(x)=\frac{1}{4+2011x} \forall x\in (0;+\infty)$

$f'(x)=\frac{-2011}{(4+2011x)^2}<0 \Rightarrow f(x)$ nghịch biến trên $(0;+\infty)$

Mà $x_1<x_2 \Rightarrow f(x_1)>f(x_2)  \Rightarrow x_2 > x_3 .... \Rightarrow x_{n+1}<x_n$ hay dãy $x_n$ đơn điệu giảm.

Kết hợp với $(*)$ thì ta suy ra $x_n $ hội tụ. 

Gọi $L  (0<L<1)$ là giới hạn của dãy $x_n$, ta có: 

$L=\frac{1}{4+2011L}$ hay $L=\frac{-2+\sqrt{2015}}{2011}$


                          "Số nguyên tố là để nhân chứ không phải để cộng."
                                                                                                                       Lev Landau

#5
LNH

LNH

    Bất Thế Tà Vương

  • Hiệp sỹ
  • 581 Bài viết

$\boxed{\text{Bài 7 (6đ)}}$

 

Các ô vuông của một bảng vuông kích thước $10x10$ được tô bởi các màu trắng hoặc đen sao cho trên mỗi hàng cũng như trên mỗi cột đều có đúng 3 ô được tô màu đen. Chẳng hạn như hình vẽ :

                                                                                10644932_634592509991520_663527048267035

Chứng minh rằng trong mọi cách tô như vậy ta luôn có thể tìm ra 10 ô được tô màu đen sao cho không có hai ô nào nằm trên cùng một hàng hay trên cùng một hàng cột.

=======Hết=======

 

Ta xét đồ thị lưỡng phân $G=\left ( A,B,E \right )$, trong đó $A$ là tập hợp các đỉnh biểu thị các hàng, $B$ là tập hợp các đỉnh biểu thị các cột. Hai đỉnh được nối với nhau khi và chỉ khi hàng và cột tương ứng giao nhau tại một ô được tô màu.

Gọi $S\subset A$ là tập con các đỉnh thuộc $A$ và $N\left ( S \right )$ là tập hợp các đỉnh thuộc $B$ mà kề với một trong các đỉnh thuộc $S$

Số các ô đen của các hàng có đỉnh thuộc $S$ là $3\left | S \right |$

Vì mỗi cột chứa $3$ ô đen nên $\left | N\left ( S \right ) \right |\geq \left | S \right |$

Theo tiêu chuẩn Hall thì tồn tại một ghép cặp hoàn hảo từ $A$ đến $B$, suy ra đpcm.


Bài viết đã được chỉnh sửa nội dung bởi LNH: 17-09-2014 - 20:23


#6
LNH

LNH

    Bất Thế Tà Vương

  • Hiệp sỹ
  • 581 Bài viết

$\boxed{\text{Bài 6 (7đ)}}$

Với mỗi số nguyên dương n, gọi $f(n)$ là số cách thay các dấu $"\pm "$ trong biểu thức $\pm 1\pm 2\pm 3...\pm n$ bởi các dấu $"+"$ hoặc $"-"$ sao cho tổng đại số nhận được bằng 0. Chứng minh rằng:

a) $f(n)=0$ khi $n\equiv 1 (mod 4)$ hoặc $n\equiv 2 (mod 4)$

b)$2^{\frac{n}{2}-1}\leq f(n)<2^n-2^{\left [ \frac{n}{2} \right ]+1}$ khi $n\equiv 0 (mod 4)$ hoặc $n\equiv 3 (mod 4)$

 

 

a) Nhận xét: Khi thay dấu cộng thành trừ (và ngược lại), tính chẵn lẻ của tổng không đổi.

Với $n\equiv 1;2\left ( mod 4 \right )$ thì tổng $1+2+...+n$ lẻ

Vì vậy, $f\left ( n \right )=0$ khi $n\equiv 1;2\left ( mod 4 \right )$

b) Chứng minh $f\left ( n \right )\geq 2^{\frac{n}{2}-1}$

Với $n=3$ và $n=4$, mệnh đề trên hiển nhiên đúng.

Ta chứng minh rằng nến $n$ đúng thì $n+4$ cũng đúng

Xét dấu của các số $n+1;n+2;n+3;n+4$:

Nếu ta đặt các dấu là $\left ( n+1 \right )-\left ( n+2 \right )-\left ( n+3 \right )+\left ( n+4 \right )$ hoặc $-\left ( n+1 \right )+\left ( n+2 \right )+\left ( n+3 \right )-\left ( n+4 \right )$ và các dấu còn lại đặt như các bộ $f\left ( n \right )$ thì ta có bộ mới thỏa yêu cầu đề bài.

Đối với các bộ thuộc $f\left ( n \right )$ có $-1$ thì ta đổi thành$+1$ rồi thêm $\left ( n+1 \right )-\left ( n+2 \right )+\left ( n+3 \right )-\left ( n+4 \right )$ thì có được bộ mới

Đối với các bộ thuộc $f\left ( n \right )$ có $+1$ thì ta đổi thành$-1$ rồi thêm $-\left ( n+1 \right )+\left ( n+2 \right )-\left ( n+3 \right )+\left ( n+4 \right )$ thì có được bộ mới

Đối với các bộ thuộc $f\left ( n \right )$ có $-2$ thì ta đổi thành$+2$ rồi thêm $\left ( n+1 \right )+\left ( n+2 \right )-\left ( n+3 \right )-\left ( n+4 \right )$ thì có được bộ mới

Đối với các bộ thuộc $f\left ( n \right )$ có $+2$ thì ta đổi thành$-2$ rồi thêm $-\left ( n+1 \right )-\left ( n+2 \right )+\left ( n+3 \right )+\left ( n+4 \right )$ thì có được bộ mới

Vậy $f\left ( n+4 \right )\geq 4f\left ( n \right )\geq 2^{\frac{n+4}{2}-1}$

Suy ra đpcm

Chứng minh $f\left ( n \right )<2^n-2^{\left [ \frac{n}{2} \right ]+1}$

Gọi $g\left ( n \right )$ là số các bộ dấu sao cho tổng trên khác $0$

Ta cần chứng minh $g\left ( n \right )> 2^{\left [ \frac{n}{2} \right ]+1}$

Chứng minh bằng qui nạp

Với $n=3$ và $n=4$ thì ta có mệnh đề trên đúng.

Ta chứng minh với $n$ đúng thì $n+4$ cũng đúng.

Xét dấu của các số $n+1;n+2;n+3;n+4$

Đối với các bộ thuộc $f\left ( n \right )$, ta đặt dấu các số $n+1;n+2;n+3;n+4$ để tổng đại số chúng khác $0$. Có $14$  cách đặt dấu cho mỗi bộ như vậy.

Đối với các bộ thuộc $g\left ( n \right )$, ta đặt dấu các số $n+1;n+2;n+3;n+4$ để tổng đại số chúng bằng $0$. Có $2$ cách đặt như vậy cho mỗi bộ

Suy ra $g\left ( n+4 \right )\geq 14f\left ( n \right )+2g\left ( n \right )>14.2^{\frac{n}{2}-1}+2.2^{\left [ \frac{n}{2} \right ]+1}>2^{\left [ \frac{n+4}{2} \right ]+1}$

Vậy ta có đpcm



#7
19kvh97

19kvh97

    Sĩ quan

  • Thành viên
  • 423 Bài viết

Bài 1: Bằng qui nạp dễ dàng chứng minh $0< x_n <1 \forall n\ge 0       (*)$

Ta cần chứng minh dãy $x_n$ đơn điệu tăng hoặc giảm.

Xét hàm số $f(x)=\frac{1}{4+2011x} \forall x\in (0;+\infty)$

$f'(x)=\frac{-2011}{(4+2011x)^2}<0 \Rightarrow f(x)$ nghịch biến trên $(0;+\infty)$

Mà $x_1<x_2 \Rightarrow f(x_1)>f(x_2)  \Rightarrow x_2 > x_3 .... \Rightarrow x_{n+1}<x_n$ hay dãy $x_n$ đơn điệu giảm.

Kết hợp với $(*)$ thì ta suy ra $x_n $ hội tụ. 

Gọi $L  (0<L<1)$ là giới hạn của dãy $x_n$, ta có: 

$L=\frac{1}{4+2011L}$ hay $L=\frac{-2+\sqrt{2015}}{2011}$

còn số hạng tổng quát nữa bạn ơi :P



#8
TonnyMon97

TonnyMon97

    Trung sĩ

  • Thành viên
  • 124 Bài viết

còn số hạng tổng quát nữa bạn ơi :P

Số hạng tổng quát có công thức tìm cho dạng này rồi nên mình không làm. ;) nhưng mà số hơi xấu @@ 


                          "Số nguyên tố là để nhân chứ không phải để cộng."
                                                                                                                       Lev Landau

#9
tuankiettvk

tuankiettvk

    Lính mới

  • Thành viên
  • 2 Bài viết

Số hạng tổng quát có công thức tìm cho dạng này rồi nên mình không làm. ;) nhưng mà số hơi xấu @@ 

công thức j zậy post lên lun đi  :lol:



#10
phamxuanvinh08101997

phamxuanvinh08101997

    Trung sĩ

  • Thành viên
  • 141 Bài viết

Ta xét đồ thị lưỡng phân $G=\left ( A,B,E \right )$, trong đó $A$ là tập hợp các đỉnh biểu thị các hàng, $B$ là tập hợp các đỉnh biểu thị các cột. Hai đỉnh được nối với nhau khi và chỉ khi hàng và cột tương ứng giao nhau tại một ô được tô màu.

Gọi $S\subset A$ là tập con các đỉnh thuộc $A$ và $N\left ( S \right )$ là tập hợp các đỉnh thuộc $B$ mà kề với một trong các đỉnh thuộc $S$

Số các ô đen của các hàng có đỉnh thuộc $S$ là $3\left | S \right |$

Vì mỗi cột chứa $3$ ô đen nên $\left | N\left ( S \right ) \right |\geq \left | S \right |$

Theo tiêu chuẩn Hall thì tồn tại một ghép cặp hoàn hảo từ $A$ đến $B$, suy ra đpcm.

Tiêu chuẩn Hall là gì nhỉ


                   :ukliam2: Đã đọc bài thì đừng tiếc gì nút Like :ukliam2:

 

:ukliam2: Không ngừng vươn xa :ukliam2:


#11
thukilop

thukilop

    Thượng sĩ

  • Thành viên
  • 291 Bài viết

Bài 3: 

10479688_641630792621025_679351473114058

*) Do $\angle CAO = \angle OCD$ nên A,I,O thẳng hàng suy ra AD cũng là tiếp tuyến của (O)

Ta có: $\angle ACD = \angle CBD$, $\angle CAD=\angle DCB$ nên $\Delta DAC \sim \Delta DCB$ vì vậy $\angle ADC= \angle CDB$ 

Suy ra: $ \angle AGE= \angle CEB= \angle AEG$ nên $\Delta AEG $ cân tại A

Hơn nữa: $ \angle ADB= \angle ADC= \angle ACD= \angle CBD$ => $\Delta CDB$ cân tại C và $\Delta CDB$ đồng dạng $ \Delta AGE$

Suy ra: $ \angle  DCB = \angle GAE= \angle GCF$ <=> $ \angle GCF= \angle DCB= \angle CGD$ => GCFD là hình thang cân

*) Từ đó ta có: $ \angle GED= \angle ECD + \angle EDC = \angle ACD = \angle ADC= \angle ADE +  \angle EDC =  \angle ADE +  \angle ADG = \angle EDG$ 

Suy ra $\Delta GED$ cân tại G cho nên $GD=GE$ mà $OE=OD$ suy ra OG là trung trực ED (1)

*) Tương tự có: $\Delta CEF $ cân suy ra $CE=CF=GD=GE$ suy ra E là trung điểm GC. Hơn nữa H là giao điểm GD với CF,CGDF là hình thang cân => EH là trung trực DF (2)

*) (1) và (2) suy ra đpcm


-VƯƠN ĐẾN ƯỚC MƠ-


#12
pdtienArsFC

pdtienArsFC

    Trung sĩ

  • Thành viên
  • 133 Bài viết

công thức j zậy post lên lun đi  :lol:

Thế này nhé:

Gọi a;b là nghiệm của PT: $2011x^2-4x-1=0.

Ta có

 $x_{n+1}+a=\frac{1}{2011x_n+1}+a=\frac{2011a(x_n+a)-2011a^2+4a+1}{2011x_n+1}=\frac{2011a(x_n+a)}{2011x_n+1}$

$\Leftrightarrow \frac{x_{n+1}+a}{x_n+a}=\frac{2011a}{2011x_n+1}$

Tương tự: $\Leftrightarrow \frac{x_{n+1}+b}{x_n+b}=\frac{2011b}{2011x_n+1}$

Do đó: $\frac{x_{n+1}+a}{x_{n+1}+b}=\frac{x_n+a}{x_n+b}.\frac{a}{b}=...$


                           80b68e1e79774daab705a98543684359.0.gif

 


#13
phatthemkem

phatthemkem

    Trung úy

  • Thành viên
  • 910 Bài viết

Bài 1: Bằng qui nạp dễ dàng chứng minh $0< x_n <1 \forall n\ge 0       (*)$

Ta cần chứng minh dãy $x_n$ đơn điệu tăng hoặc giảm.

Xét hàm số $f(x)=\frac{1}{4+2011x} \forall x\in (0;+\infty)$

$f'(x)=\frac{-2011}{(4+2011x)^2}<0 \Rightarrow f(x)$ nghịch biến trên $(0;+\infty)$

Mà $x_1<x_2 \Rightarrow f(x_1)>f(x_2)  \Rightarrow x_2 > x_3 .... \Rightarrow x_{n+1}<x_n$ hay dãy $x_n$ đơn điệu giảm.

Kết hợp với $(*)$ thì ta suy ra $x_n $ hội tụ. 

Gọi $L  (0<L<1)$ là giới hạn của dãy $x_n$, ta có: 

$L=\frac{1}{4+2011L}$ hay $L=\frac{-2+\sqrt{2015}}{2011}$

Hàm nghịch biến thì suy ra hai dãy con với chỉ số chẵn và lẻ với tính đơn điệu trái ngược nhau  :( 


  Hầu hết mọi người đều chấp nhận thua cuộc ngay khi họ sắp thành công. Họ dừng lại

 

ngay trước vạch đích, cách chiến thắng chỉ một bàn chân” -H. Ross Perot

 

  “Tránh xa những kẻ coi nhẹ tham vọng của bạn. Những kẻ nhỏ nhen luôn như thế, còn

 

những người thực sự vĩ đại sẽ khiến bạn cảm thấy rằng bạn cũng có thể trở nên vĩ đại”

 

-Mark Twain

:botay :like :icon10: Huỳnh Tiến Phát ETP :icon10: :like :botay

$WELCOME$ $TO$ $MY$ $FACEBOOK$: https://www.facebook.com/phat.huynhtien.39


#14
TonnyMon97

TonnyMon97

    Trung sĩ

  • Thành viên
  • 124 Bài viết

Hàm nghịch biến thì suy ra hai dãy con với chỉ số chẵn và lẻ với tính đơn điệu trái ngược nhau  :( 

Đúng rồi tớ xin lỗi nhé. Hồi đó xem vội nên làm nhằm. Thêm bước CM 2 dãy chẵn-lẽ có cùng giới hạn :P


                          "Số nguyên tố là để nhân chứ không phải để cộng."
                                                                                                                       Lev Landau

#15
toanhoc2017

toanhoc2017

    Thiếu úy

  • Banned
  • 628 Bài viết

a) Nhận xét: Khi thay dấu cộng thành trừ (và ngược lại), tính chẵn lẻ của tổng không đổi.
Với $n\equiv 1;2\left ( mod 4 \right )$ thì tổng $1+2+...+n$ lẻ
Vì vậy, $f\left ( n \right )=0$ khi $n\equiv 1;2\left ( mod 4 \right )$
b) Chứng minh $f\left ( n \right )\geq 2^{\frac{n}{2}-1}$
Với $n=3$ và $n=4$, mệnh đề trên hiển nhiên đúng.
Ta chứng minh rằng nến $n$ đúng thì $n+4$ cũng đúng
Xét dấu của các số $n+1;n+2;n+3;n+4$:
Nếu ta đặt các dấu là $\left ( n+1 \right )-\left ( n+2 \right )-\left ( n+3 \right )+\left ( n+4 \right )$ hoặc $-\left ( n+1 \right )+\left ( n+2 \right )+\left ( n+3 \right )-\left ( n+4 \right )$ và các dấu còn lại đặt như các bộ $f\left ( n \right )$ thì ta có bộ mới thỏa yêu cầu đề bài.
Đối với các bộ thuộc $f\left ( n \right )$ có $-1$ thì ta đổi thành$+1$ rồi thêm $\left ( n+1 \right )-\left ( n+2 \right )+\left ( n+3 \right )-\left ( n+4 \right )$ thì có được bộ mới
Đối với các bộ thuộc $f\left ( n \right )$ có $+1$ thì ta đổi thành$-1$ rồi thêm $-\left ( n+1 \right )+\left ( n+2 \right )-\left ( n+3 \right )+\left ( n+4 \right )$ thì có được bộ mới
Đối với các bộ thuộc $f\left ( n \right )$ có $-2$ thì ta đổi thành$+2$ rồi thêm $\left ( n+1 \right )+\left ( n+2 \right )-\left ( n+3 \right )-\left ( n+4 \right )$ thì có được bộ mới
Đối với các bộ thuộc $f\left ( n \right )$ có $+2$ thì ta đổi thành$-2$ rồi thêm $-\left ( n+1 \right )-\left ( n+2 \right )+\left ( n+3 \right )+\left ( n+4 \right )$ thì có được bộ mới
Vậy $f\left ( n+4 \right )\geq 4f\left ( n \right )\geq 2^{\frac{n+4}{2}-1}$
Suy ra đpcm
Chứng minh $f\left ( n \right )<2^n-2^{\left [ \frac{n}{2} \right ]+1}$
Gọi $g\left ( n \right )$ là số các bộ dấu sao cho tổng trên khác $0$
Ta cần chứng minh $g\left ( n \right )> 2^{\left [ \frac{n}{2} \right ]+1}$
Chứng minh bằng qui nạp
Với $n=3$ và $n=4$ thì ta có mệnh đề trên đúng.
Ta chứng minh với $n$ đúng thì $n+4$ cũng đúng.
Xét dấu của các số $n+1;n+2;n+3;n+4$
Đối với các bộ thuộc $f\left ( n \right )$, ta đặt dấu các số $n+1;n+2;n+3;n+4$ để tổng đại số chúng khác $0$. Có $14$ cách đặt dấu cho mỗi bộ như vậy.
Đối với các bộ thuộc $g\left ( n \right )$, ta đặt dấu các số $n+1;n+2;n+3;n+4$ để tổng đại số chúng bằng $0$. Có $2$ cách đặt như vậy cho mỗi bộ
Suy ra $g\left ( n+4 \right )\geq 14f\left ( n \right )+2g\left ( n \right )>14.2^{\frac{n}{2}-1}+2.2^{\left [ \frac{n}{2} \right ]+1}>2^{\left [ \frac{n+4}{2} \right ]+1}$
Vậy ta có đpcm

Bài hay




1 người đang xem chủ đề

0 thành viên, 1 khách, 0 thành viên ẩn danh